GMAT Ninja Quant Ep 5: Word Problems

Поділитися
Вставка
  • Опубліковано 19 жов 2024

КОМЕНТАРІ • 38

  • @KrazedAspects
    @KrazedAspects Місяць тому +2

    Really struggled with these questions for some reason, so it was helpful with the way you broke these down. Thanks !

    • @GMATNinjaTutoring
      @GMATNinjaTutoring  Місяць тому

      Thank you so much for watching, and have fun studying!

  • @vietchinhnguyen4765
    @vietchinhnguyen4765 Місяць тому +3

    For Q5, not sure if the reasoning I used is 'appropriate' but here I did it as follows:
    - the {x} part blah blah already scared me shitless so I turn a blind eye, set it aside, and get on with the rest of the question stem.
    - we are given p hours and intervals of 15 min, so I break the p hours into each interval of 15 min as p*60/15
    >> The cost of the video should be: 140 + 20(p*60/15-1) (Reasoning: we have in total p*60/15 interval, the first interval cost $140; the rest cost 20 each)
    Simplify the equation, we have cost of the video as: 120 + 80p. Looking in all the option, we can choose A as it is closest to our calculation.
    I guess this way wouldn't work if the additional minutes (from after the 1st 15 min) can not be broken into equal intervals

    • @happyhummer5889
      @happyhummer5889 День тому

      I used the same thinking pattern, but I sense I missed something in this process. That is I believe how to justify the {4p} part of the equation?

  • @skyler-t8s
    @skyler-t8s Місяць тому +7

    For Q5, i think you meant to circle A, not D, as the correct answer! Thanks for all of your great videos.

    • @behazyy
      @behazyy Місяць тому +1

      was about to ask about this haha he simplified it to 120 + 20{4P} then circled 140 lol

    • @davidvigilius8300
      @davidvigilius8300 Місяць тому

      Another great example of how rushing leads to problems on the GMAT

    • @GMATNinjaTutoring
      @GMATNinjaTutoring  Місяць тому +8

      ARGH! You're right - I did mean to circle (A) instead of (D)!
      As @davidvigilius8300 says in his comment, this is a really good example of how any sort of rushing or not giving the question all of your attention can lead to unnecessary slip ups.
      Thank you for pointing this out!

    • @sichunwang1337
      @sichunwang1337 Місяць тому

      OK thanks. I hated the moment that I thought I understand and realized that I was wrong

  • @jadmouazen7657
    @jadmouazen7657 Місяць тому +5

    I just have a question: The average time for answering any quant question is 2mins 9 seconds if i am not mistaken, but it takes the instructor 6 mins+ sometimes to solve it. I know there is explanation time, but for questions like question 6 it is very intimidating that solving such questions on the GMAT will take this much time, especially with the stress and time constraints.
    Any advice? Love your videos but i noticed this is a common theme.

    • @GarethNihongo
      @GarethNihongo 26 днів тому

      My take on this is that the instructor is Not skipping any step solving an equation, whereas we will absolutely do so in the exam

  • @AnshumanNarain
    @AnshumanNarain Місяць тому +1

    Hey, thank you for your extremely valuable videos
    On quant I always try to 'push the question' and just that thinking has helped me a lot
    Giving my GMAT in Jan 2025, thank you once again

    • @GMATNinjaTutoring
      @GMATNinjaTutoring  Місяць тому

      Thank you so much, and enjoy the next several months of studying!

    • @sparshsinghal7307
      @sparshsinghal7307 14 днів тому

      If you give your GMAT in 2025, can you still do your MBA in 2025 if you get a decent score or you have to take a drop for 1 year?

  • @VidurSeth-b7o
    @VidurSeth-b7o Місяць тому +2

    Hey, thanks so much for these videos - really grateful for them.
    I had a question regarding Q6. I didn't see anything in the question that stated we needed to consider whole days (though I could be mistaken). Based on that, I didn't feel it was necessary to test scenarios with 4, 3, 2, 1, or 0 days, as half days are also a possibility, and people often get paid for them. Moreover, the assumption that the GMAT doesn't typically expect highly detailed calculations led me to this approach. As a result, I arrived at answer choice C. I'd love to hear your thoughts on this. Do you think this approach makes sense?

    • @gayathri186
      @gayathri186 20 днів тому

      Hey I thought the same and got C as option . However I think question gives daily rate ...so.day a unit ,.cannot have half day etc . May b this is why

    • @EhabMAhmed
      @EhabMAhmed 20 днів тому

      That’s exactly what I did and what I got when solving the question at first glance. From the first point, I calculated the number of days for B, and from the second point, I calculated the number of days for C. By combining them, I should have been able to determine the number of days for A. However, my mistake was that I got 5 days for A, even though it was mentioned that A should have fewer than 5 days. This is the only part that confused me. But based on my initial impression, I would choose C

  • @sparshsinghal7307
    @sparshsinghal7307 14 днів тому +1

    Q5 seemed so much harder than Q6, Q6 was relatively very easy.

  • @6uinnn
    @6uinnn Місяць тому

    Hey y'all, was just wondering if you're going to be going over and remake every video from the GMAT Quant playlist or only a select few of them?

    • @GMATNinjaTutoring
      @GMATNinjaTutoring  Місяць тому +2

      We're remaking everything in the quant playlist. In most cases, the remakes aren't really an overhaul of the content -- relatively little has changed in terms of the actual quant content of the GMAT, other than the removal of geometry. But in the older videos, we use outdated language -- for example, we refer to the old score scale, or treat DS as part of quant -- so we're refreshing all of the videos.
      It'll take us another month or two to get them all filmed, edited, and posted. In the meantime, it's totally fine to watch the old ones -- all of the content is still completely relevant for the current version of the GMAT.

  • @nabidishtiaque9303
    @nabidishtiaque9303 3 дні тому

    Q5
    that equation you made 20{4p}+140-20 I didn't understand at all
    where did the 4p came from? 🙂

  • @shckwave
    @shckwave Місяць тому

    For Q6, they mentioned that the salary was at a “daily rate”, but the 11,300 was a “monthly” number. Wouldn’t you need to include - times 30 or 31 (based on the number of days in a month) into the equations?

    • @GMATNinjaTutoring
      @GMATNinjaTutoring  Місяць тому

      No, we don't need to multiply the 11,300 by anything. Brian will work a certain number of days in the month and will earn the daily rate listed for each day, depending on which company he works for. We can then sum the total amount he earns for working those days, which will equal the $11,300 he earned last month.
      I hope that helps!

  • @sparshsinghal7307
    @sparshsinghal7307 14 днів тому

    for q5 I was worried about the {x} part so I skipped it, and then I wrote the equation while assuming p=1 hr
    this gave me=140$ + 20(3), 20(3) as p is 1, so 45min=3 blocks
    I got the result 200$
    then I just plugged in {p} as 1 everywhere and chose where it gave me 200$ as a result, which was A
    is this an alternative approach? I feel this is a very wrong way of doing it

  • @himakshibatra
    @himakshibatra Місяць тому

    Q5, I assumed that p = 2 hrs which gives us 8 (15 min) periods. Ideally, the editing then would cost 140+20(7)= 280. Putting values in Option A: 120+20{4p} = 120+20{4*2} = 120+20*8= 120+160 = 280. Option A would be right. Is this an okay method? Because {2}=2, braces don't seem to cause a problem.

    • @GMATNinjaTutoring
      @GMATNinjaTutoring  Місяць тому +1

      Your method has worked with this set of answer choices, but it wouldn't work if one of the answer choices had been 120 + 80{p}.
      In the case shown in the video with p = 2.3, the correct charge would be given by 120 + 20{4p} = 320 and 120 + 80{p} would give 360. However, using p = 2, we'd get 120 + 20{4p} = 280 and 120 + 80{p} = 280, so we wouldn't be able to tell these two answers apart.
      When we use numbers to check specific conditions, we've got to be careful we stretch those conditions as far as possible. Using an integer number of hours for p in this question is risky because the conditions only really change when using a duration such as 2 hours 20 minutes, 3 hours 40 minutes, or something similar.
      I hope that helps!

    • @himakshibatra
      @himakshibatra Місяць тому

      @@GMATNinjaTutoring Thanks

  • @bhushanshaligram1995
    @bhushanshaligram1995 25 днів тому

    In question 2, why didn't we substitute 3s = s+6 and then solved the question from there? In that case, both the statements were required to come to conclusion.

    • @GMATNinjaTutoring
      @GMATNinjaTutoring  25 днів тому

      We only need to combine the two statements if neither of them is sufficient to answer the question on their own. In this case, statement 1 was sufficient and statement 2 was insufficient to answer the question so the answer was (A).
      For more on how data sufficiency questions work, check out the link below.
      I hope that helps!
      www.mba.com/exams-and-exam-prep/gmat-exam-prep/gmat-data-sufficiency-expert-tips

  • @lisacassama54
    @lisacassama54 Місяць тому

    For Q1, shouldn't it be 3(A+5)=(B+5). Instead of (A+5)=3(B+5), because they say "Anneka will be 3 times as old as Badshah" ?

    • @GMATNinjaTutoring
      @GMATNinjaTutoring  Місяць тому +1

      If Anneka is three times as old as Badshah, then Anneka is the older one of the two.
      If we do 3(A+5)=(B+5), then the value of B will be greater than the value of A, which would suggest that Badshah is older than Anneka and we don't want that.
      However, if we do (A+5)=3(B+5), then the value of A will be greater than the value of B and Anneka will be older than Badshah. This is consistent with the wording of the question, so this is the way to set up the equation.
      I hope that helps!

  • @Xoustus
    @Xoustus 19 днів тому

    For Q3, I’m having a really hard time understanding how 3 weeks left became ( W-3). Could someone explain?

    • @GMATNinjaTutoring
      @GMATNinjaTutoring  17 днів тому +1

      Let's define w as the number of weeks the team had to reach their sales goal IN TOTAL. We're told the team has made 550 sales from the start of this period and has only 3 weeks left to make the final 4,200 sales. This means they've made 550 sales per week for (w - 3) weeks and needs to make a further 4,200 sales to reach their quota.
      I hope that helps!

    • @sparshsinghal7307
      @sparshsinghal7307 14 днів тому

      @@GMATNinjaTutoring so basically
      w= weeks they have in total to meet their quota
      3 = weeks they have left to make 4200 sales
      x= sales they had planned to make in total
      700w= x
      w-3 gives the amount of sales they've made so far, and to get the amount of sales they've made so far, we do (x-4200), total - sales they need to make would give us sales they've made so far, so the equation would be
      550(w-3)=x-4200
      550(w-3)+4200=x
      700w=x
      700w=550(w-3)+4200
      right?

    • @GMATNinjaTutoring
      @GMATNinjaTutoring  2 дні тому

      You've got it! That's a great way of thinking about this question.

  • @nabidishtiaque9303
    @nabidishtiaque9303 3 дні тому

    Q6
    already B C are given and they can easily get A
    so the main answer should be C together sufficient
    why did you bring C=5 C=6 here?
    why make it so complicated or is there anything I am missing?

    • @GMATNinjaTutoring
      @GMATNinjaTutoring  3 дні тому +1

      You're absolutely right that we can answer this question using the information in statements 1 and 2 combined. However, we have to check whether we can answer the question using the information in either statement by itself first. We'll only worry about combining the statements once we've checked each of them individually.
      The solution shown in the video demonstrates that the information in statement 1 is insufficient to answer the question, but the information in statement 2 is sufficient. This is why the answer to this question is (B).
      I hope that helps!

    • @nabidishtiaque9303
      @nabidishtiaque9303 3 дні тому

      @@GMATNinjaTutoring That's really eye opening 🤍